Difference between revisions of "1987 OIM Problems/Problem 5"
(solution) |
|||
(One intermediate revision by the same user not shown) | |||
Line 1: | Line 1: | ||
== Problem == | == Problem == | ||
If <math>r</math>, <math>s</math>, and <math>t</math> are all the roots of the equation: | If <math>r</math>, <math>s</math>, and <math>t</math> are all the roots of the equation: | ||
− | <cmath>x(x-2)3x-7)=2</cmath> | + | <cmath>x(x-2)(3x-7)=2</cmath> |
− | (a) Prove that <math>r</math>, <math>s</math>, and <math>t</math> are all | + | (a) Prove that <math>r</math>, <math>s</math>, and <math>t</math> are all positive. |
− | (b) Calculate | + | (b) Calculate <math>\arctan r + \arctan s + \arctan t</math>. |
− | Note: | + | Note: The range of <math>\arctan x</math> falls between <math>0</math> and <math>\pi</math>, inclusive. |
~translated into English by Tomas Diaz. ~orders@tomasdiaz.com | ~translated into English by Tomas Diaz. ~orders@tomasdiaz.com | ||
== Solution == | == Solution == | ||
− | {{ | + | (a) Consider the polynomial <math>f(x)=x(x-2)(3x-7)-2</math>. When <math>x</math> is negative, the value <math>x(x-2)(3x-7)</math> is clearly negative, so adding this to <math>-2</math> will yield negative <math>f(x)</math>; therefore, there cannot be a negative root. There also clearly cannot be a root equal to zero, so all that remains is to prove that the roots are all real. This can be achieved by by using the Intermediate Value Theorem; notice that <math>f(0)=-2,f(1)=2,f(2)=-2,</math> and <math>f(3)=4</math>, which imply three real roots, so clearly <math>r,s,t</math> are all real and are thus positive. |
+ | |||
+ | (b) Let <math>\alpha=\arctan r,\beta=\arctan s,\gamma=\arctan t</math>. Then <math>\tan\alpha=r,\tan\beta=s,\tan\gamma=t</math>. By multiple applications of the sum of tangents formula: | ||
+ | <cmath>\tan(\alpha+\beta+\gamma)=\frac{\tan\alpha+\tan\beta+\tan\gamma-\tan\alpha\tan\beta\tan\gamma}{1-\tan\alpha\tan\beta-\tan\beta\tan\gamma-\tan\gamma\tan\alpha}=\frac{r+s+t-rst}{1-rs-st-rt}</cmath> | ||
+ | If we expand <math>f(x)</math>, we find that | ||
+ | <cmath>f(x)=3x^3-13x^2+14x-2</cmath> | ||
+ | implying that | ||
+ | <cmath>r+s+t=\frac{13}{3},rs+st+rt=\frac{14}{3},rst=\frac{2}{3}</cmath> | ||
+ | Therefore, | ||
+ | <cmath>\tan(\alpha+\beta+\gamma)=\frac{\frac{13}{3}-\frac{2}{3}}{1-\frac{14}{3}}=\frac{13-2}{3-14}=-1</cmath> | ||
+ | Due to our given range of <math>\arctan x</math>, we know that <math>\alpha+\beta+\gamma=\boxed{\frac{3\pi}{4}}</math>. | ||
+ | |||
+ | ~ [https://artofproblemsolving.com/wiki/index.php/User:Eevee9406 eevee9406] | ||
== See also == | == See also == | ||
https://www.oma.org.ar/enunciados/ibe2.htm | https://www.oma.org.ar/enunciados/ibe2.htm |
Latest revision as of 17:13, 19 April 2025
Problem
If ,
, and
are all the roots of the equation:
(a) Prove that ,
, and
are all positive.
(b) Calculate .
Note: The range of falls between
and
, inclusive.
~translated into English by Tomas Diaz. ~orders@tomasdiaz.com
Solution
(a) Consider the polynomial . When
is negative, the value
is clearly negative, so adding this to
will yield negative
; therefore, there cannot be a negative root. There also clearly cannot be a root equal to zero, so all that remains is to prove that the roots are all real. This can be achieved by by using the Intermediate Value Theorem; notice that
and
, which imply three real roots, so clearly
are all real and are thus positive.
(b) Let . Then
. By multiple applications of the sum of tangents formula:
If we expand
, we find that
implying that
Therefore,
Due to our given range of
, we know that
.